Top Banner
Subject Matter Jurisdiction: What court do we go to state or federal? Two ways to get into federal court / Windows to federal court—2 forms of jurisdiction: 1. Diversity, 2. Federal question (both come from Art III Sec 2). §1332 – Diversity; §1331 – Federal Question (No 1331 until 1875, no general fed question jurisdiction only statutes.) 1. Diversity (§ 1332) suing under state law state common law claims and ∆ different state to avoid out of state prejudices to give a fair chance: requires (i) diversity in citizenship (complete diversity under strawbridge); (ii) AIC: constitution says nothing about it but §1332.(creates a filter, narrows jurisdiction.) - < $75,000 2. Federal Question (§ 1331) second door to get into federal court; well-pleaded complaint rule - Two kinds of cases the fed ct. won’t hear: family law, (divorces etc. even after meeting AIC) and general probate cases (will, trust) even where the requirements in div. are met. A. State Court Cases: Family law, no state diversity, AIC < $75, 000 C. Federal Court Cases: ex. Patent law – exclusive area of federal jurisdiction. B. Overlap of A & C: Fed. Question claim can be brought to state court. - Diversity met but ∆ does not want to remove. - Challenge of SMJ is most fundamental challenge for constitutional reasons. - can be brought in anytime (can be used as a trap with SOL issues in order to stall the case.) A. Diversity of Citizenship § 1332 (a)(1) - Two requirements for a diversity citizenship case: (a) Case must be between citizens of different states; (b) Amount in controversy must exceed $75, 000. (Mas v. Perry, AIC $10,000 then) a. Case must be between citizens of different states. i. Complete diversity rule for invoking diversity jurisdiction – there is no diversity if any plaintiff is a citizen of the same state as any defendant. (Strawbridge v. Curtiss) ii. An American is a citizen of the state in which she is domiciled. Domicile Test: RESIDENCE + INTENT TO STAY b. Amount in controversy must exceed $75, 000. : Not constitutionally required, added by §. (Art.III§2 is further narrowed by 28 U.S.C. §1332(a) brings in AIC ($75, 000) governed by Strawbridge; complete diversity Diversity Test: P or ∆ are domicile where? residence + intention to stay at what point does one develop an intention to stay? have they put down roots like voter’s card, house, dl etc. - this means that even not permanent but my house for now; domicile - not purely Part I Federal Court Jurisdiction (Subject Matter Jurisdiction) Diversity of Citizenship (Diversity Jurisdiction: about the parties) Federal Question Jurisdiction (Deals with what is source of law) Supplemental Jurisdiction – Pendant , Ancilliary The issue of REMOVAL 1. For a corporation, citizenship is defined by § 1332(c)(1). Corporation is a citizen of all states where incorporated, and the state where it has its principal place of business (NERVE CENTER.) – corp. treated as persons. 2. Partnerships: Distinct, partnerships of individuals, if partners live in many states then domicile=every state where the partner lives. 1
33

Exam Outline Civ Pro

Jan 29, 2016

Download

Documents

Puruda Amit

Civ Pro Outline
Welcome message from author
This document is posted to help you gain knowledge. Please leave a comment to let me know what you think about it! Share it to your friends and learn new things together.
Transcript
Page 1: Exam Outline Civ Pro

Subject Matter Jurisdiction: What court do we go to state or federal?

Two ways to get into federal court / Windows to federal court—2 forms of jurisdiction: 1. Diversity, 2. Federal question (both come from Art III Sec 2). §1332 – Diversity; §1331 – Federal Question (No 1331 until 1875, no general fed question jurisdiction only statutes.)

1. Diversity (§ 1332) suing under state law state common law claims ∏ and ∆ different state to avoid out of state prejudices to give a fair chance: requires (i) diversity in citizenship (complete diversity under strawbridge); (ii) AIC: constitution says nothing about it but §1332.(creates a filter, narrows jurisdiction.) - < $75,0002. Federal Question (§ 1331) second door to get into federal court; well-pleaded complaint rule

- Two kinds of cases the fed ct. won’t hear: family law, (divorces etc. even after meeting AIC) and general probate cases (will, trust) even where the requirements in div. are met.

A. State Court Cases: Family law, no state diversity, AIC < $75, 000C. Federal Court Cases: ex. Patent law – exclusive area of federal jurisdiction.B. Overlap of A & C: Fed. Question claim can be brought to state court.

- Diversity met but ∆ does not want to remove.- Challenge of SMJ is most fundamental challenge for constitutional reasons. - can be brought in anytime (can be used as a trap with SOL issues in order to stall the case.)

A. Diversity of Citizenship

§ 1332 (a)(1) - Two requirements for a diversity citizenship case: (a) Case must be between citizens of different states; (b) Amount in controversy must exceed $75, 000. (Mas v. Perry, AIC $10,000 then)

a. Case must be between citizens of different states.i. Complete diversity rule for invoking diversity jurisdiction – there is no diversity if any plaintiff is a citizen of the same state as any

defendant. (Strawbridge v. Curtiss) ii. An American is a citizen of the state in which she is domiciled. Domicile Test: RESIDENCE + INTENT TO STAY

b. Amount in controversy must exceed $75, 000. : Not constitutionally required, added by §. (Art.III§2 is further narrowed by 28 U.S.C. §1332(a) brings in AIC ($75, 000) governed by Strawbridge; complete diversity

Diversity Test: P or ∆ are domicile where? residence + intention to stay at what point does one develop an intention to stay? have they put down roots like voter’s card, house, dl etc. - this means that even not permanent but my house for now; domicile - not purely objective, court looks at transitions, where are they moving, plan to move etc. Mas v. Perry - res. with intention to stay Mrs. Mas had left Miss. , even then didn't lose Miss res. until went somewhere

Rule of Complete diversity is required with multiple parties:- No P from same state as any of the D - strawbridge case complete div not constitutionally required but court in 1886 read complete diversity into the statute.- Business entities 1332(c): corporations: state of incorporation and principle place of business. - NERVE CENTRE THEORY (the brain, where decisions are made)- Partnerships and other entities: 1332 doesn't speak, has been interpreted: domiciled in every place where their partners are domiciled.

- Aliens: 1332 a2, a3,c1 : If someone is here legally with green card they are domiciled in the state where they reside, PR are treated as residents of that state 1332a2 makes clear that P and D some of whom are aliens living here; ex P from IL suing British PR residing in IN - can be sued under 1332a2, but can’t sue a PR residing in IL 1332a3: aliens not PR here.- American citizen + British National can sue Australian National (as long as any one side has a US Citizen).

One exception to complete div = class action, 1332d (provides for how class actions are treated) - congress passed class action fairness act (10

Part IFederal Court Jurisdiction (Subject Matter Jurisdiction)

Diversity of Citizenship (Diversity Jurisdiction: about the parties) Federal Question Jurisdiction (Deals with what is source of law) Supplemental Jurisdiction – Pendant , Ancilliary The issue of REMOVAL

1. For a corporation, citizenship is defined by § 1332(c)(1). Corporation is a citizen of all states where incorporated, and the state where it has its principal place of business (NERVE CENTER.) – corp. treated as persons.2. Partnerships: Distinct, partnerships of individuals, if partners live in many states then domicile=every state where the partner lives.3. Family law (ex. Divorce) and Probate (ex. Will) – won’t be accepted in fed. courts; fall at the heart of state cons. sov.

1

Page 2: Exam Outline Civ Pro

years ago) : prevents minimal diversity as minimal diversity is all you need. Amt. in C. = $5mill. (all aggregated claims)

Aggregating claims:

P— $30,000— D + another $50,000 claim, can aggregate but what cannot happen is two plaintiffs joining the claims and aggregating Separate plaintiffs can’t aggregate - will be the end run around the controversy.

Suppose there are two defendants: D1 and D2, can P get into fed court by aggregating the claims of these two Ds - NO- Each party has to come in on his own against a particular defense. Can’t bootstrap by suing a second D.

DIVERSITY OF CITIZENSHIP §1332

Diversity Test – Domicile (Citizen is qualified with the common law concept of “Domicile” = residence (open ended, no indefinite intent to leave) + intent to stay indefinitely.)Aggregation: Individual P can aggregate multiple claims against an individual D to reach AIC, can add claims together using Supp. J. 2 Ps cannot aggregate their individual claims against same D If one P satisfied 75K, others can piggy back along- Ex. ∏1 - $45,000 (tort claim) + $40, 000 (contract claim) = $85, 000 > $75, 000 but could not have been combined if two different ∏s (in order to sue out of state ∆.)- Monetary requirement adds a filter. AIC creates a filter, narrows jurisdiction.

o The court decides the AIC in good faith, court looks at whether the claim was brought in good faith. HYPO: IN ∏ suing IL and IN ∆, would diversity be met?NO Minimal Diversity: at least one ∆ is from a different state than ∏. (does not hold right now) Strawbridge v. Curtis: requires complete diversity: No party on ∏ side can be from the same state as ∆ (further narrows fed. j., not reqd. by art.III but framers of the §.) – Not clear in the constitution or §1332 -- because of interpretation of jurisdiction (was read by SCOTUS as such.) Hence, CD not constitutionally requd. But interpretation of §1332. (holds right now)- If out of state ∏ suing instate ∆, can create bias, hence Div. J – Fed. Law – to get ∏ out of state court. (States used to be separate sovereigns, hence, prejudices, hence div. j. was created.)

B. FEDERAL QUESTION JURISDICTION § 1331

Federal Question: “Arises Under” – 1331-District courts shall have original jurisdiction of all civil actions arising under the Constitution, laws, or treatises of the US (federal laws). Must be part of P’s actions (can’t be in anticipation of a defense)

o Well pleaded complaint rule-does a federal question arise on the face of the complaint (can’t be in anticipation of responses/defenses).

- Second door to get into fed ct.- Mottley case made clear that fed question has to be on the face of P’s well pleaded complaint All depends on P’s complaints; D can’t remove in defense.Wrinkles: P might have state issues but if significant fed. component P might be about to get in; if state law claim depends upon fed q then can get it Grable/Merrel Dow: Fed Q doesn't have to be the cause of action. Could be lying at the back of P’s state cause of action. - The bonds issued were unconstitutional

Mas v. Perry1332(a)(2) applies here: If the amount in controversy is greater than $75,000, then yes, 28 U.S.C. § 1332(a)(2) allows diversity jurisdiction over a case where one party is a U.S. citizen and the other is a foreign citizen.Facts: French man & Mississippi woman, married in Mississippi, moved to Louisiana to study. In Louisiana, Perry had been observing them through two-way mirrors in the apartment. The plaintiffs sued, during the trial, defendant objected to the court’s SMJ, claiming there was no diversity between the parties. The plaintiffs won, defendant appealed to the 5th Circuit solely on jurisdictional grounds.Issue: Perry says, case doesn’t belong in federal court?Analysis: Mr. Mas, still a citizen of France at time of filing, Mrs. Mas domiciled in Mississippi. Mrs. Mas lived in Mississippi when she married Mr. Mas, and her domicile did not change, they moved to Louisiana, but only as students (even though no intention of returning to Miss.The court refuses to apply the general rule that a wife is domiciled where her husband is domiciled to the case where an American woman marries a foreign man. The court cites a statute, 8 U.S.C. § 1489, which says that an American woman does not lose her citizenship if she marries a foreign man.Court favored wife (even though no intention of returning to Miss, kept coming back to LA (change of domicile if taking up res. In diff. domicile with intention to remain there) mainly because of husband, cannot do this end of the run today.-Argue against her by looking at what else she applied for, for ex. DL, voter’s card?;out of state/in state tuition clause.

2

Page 3: Exam Outline Civ Pro

No diversity, P’s case of action was based on breach of contract, which is generally a state claim based on a common law No FQJ- It’s a retroactive case, no new passes should be issued, this is 35 years old. Should not be applied retroactively. If this applies, then it’s a 5th amendment issue, hence, Federal. (state courts could have been more sympathetic of Ps in circuit court.) federal question must arise from the plaintiff’s original cause of action -- Even though it is now a federal issue but originally wasn’t. Initially was a state law claim, not a federal one, hence, SCOTUS rejected. (Cannot be based on ∆’s anticipated defense) §1331 provides statutory basis but it is about civil action arising under P’s complaint, not about what ∆ raises in defense.

WELL PLEADED COMPLAINT RULE: o Federal question must arise on the face of a well pleaded complaint, cannot arise of the face of defense (anticipated/later), has to be

the basis for P’s original cause of action (has to be founded on federal law.)o The well-pleaded complaint rule talks only about the complaint . We look for a federal question on the face of the complaint. If the

federal complaint, as filed by the plaintiff, does not state the federal question, then there is no federal question. o Why are anticipated defenses not good enough? It’s related to the efficiency argument. A good lawyer can make a lot of different

federal arguments that the defense could make. But the other side might not make that argument. In fact, sometimes they will strategically not make that argument.

In Mottley, § 1257 (to get into Supreme Court). (§1250’s -- §1257} permits appeal – broader jurisdiction – in situations where you cannot originally get through HOW TO REACH SCOTUS FOR II TIME By using other §s even if not §1331. – If you can prove min. div. in certain cases, Congress has power to permit FQJ.- Ps case was contract and RR response was congressional statute §1257: permits an appeal to the scotus if fed issue. Mottley says you can’t get into fed dist. Ct originally on d’s defense but 1257 is response to that What if state supreme court un-sympathetic; 1257 permits appeal from state supreme court to SCOTUS.

26 USCS § 6335 (a); This case warranted federal jurisdiction because the owners’ claim was premised on the IRS’ failure to give adequate notice as defined by federal law.

The §’s meaning belonged in federal court given the government’s strong interest in tax collection. Forum non-convenience – A discretionary power that allows courts to dismiss a case where another court or forum is much better

suited to hear the case. This dismissal does not prevent P from refiling his/her case in the more appropriate forum.Two overlapping circles:A: State Ct.C: Fed. Ct. (Div. Juris and FQJ)- Exclusive fed. jurisdictionB: - Where P has a choice to go to state federal court.

o For non-exclusive federal claim also.

Louisville & Nashville R.R. v. Mottley (SCOTUS, 1908)FACTS Mottleys got a lifetime free rail pass to settle a claim against the railroad. Later, Congress outlawed all lifetime rail passes and the railroad refused to honors the Mottleys’ pass anymore. The Mottleys sued in federal court for specific performance to keep their pass. They basically anticipated that the defense of the railroad would be to say that Congress forbade them from honoring the pass but that such an interpretation of the statute violated the Fifth Amendment, and that’s why the Mottleys brought suit in federal court. The federal court granted specific performance to the Mottleys and the railroad appealed to the Supreme Court.Issue: Did the federal district court have subject matter jurisdiction over the case? (This is the real issue, though it wasn’t one of the issues argued before the court.)Rule: A suit arises under the Constitution and laws of the United States only if the original statement of the plaintiff’s cause of action shows that it is based on the Constitution or federal statutes. (A federal court can’t have jurisdiction just because the defendant might use a federal law or the Constitution to defend himself.)HOLD: No federal jurisdiction since no federal question since it doesn’t arise on the face of the complaint – must have a well pleaded complaint. – no SMJ

GRABLE v. DARUEFacts: The IRS seized P's property to satisfy P's tax delinquency. IRS sold property to D. P did not exercise its right to redeem the property within 180 days of the sale even though P was given notice of the sale through certified mail. 5 years later, P brought a quiet title action against D and claimed that P was not notified in the exact manner required by federal statute. (personal service v. certified mail). D removed the case to Federal District Court as presenting a federal question because the claim of title depended on the interpretation of the notice statute in federal law. P sought to remand the case to state court. District Court denied and granted summary judgment to D.  PH: Circuit Ct: Judgment for D (proper jurisdiction); COA: Affirmed; SCOTUS: Affirmed, found proper FQJIssue: How can a claim involving a cause of action not started by a federal statute be tried in federal court under federal-question jurisdiction?Rule: A federal court may have jurisdiction over a state cause of action, if the action has a substantial federal component in actual controversy, and federal jurisdiction would not disrupt the balance of labor between state and federal courts.Hold: A claim that doesn't follow the "creation test" can be tried in federal court if the court decides that the state law claim necessarily raises a stated federal issue, actually disputed and substantial, which a federal forum may entertain without disturbing any congressionally approved balance of federal and state judicial responsibilities. 

3

Page 4: Exam Outline Civ Pro

RECAP:o P has federal cause of action, but brings it to state court.o Removal: Defendant can remove P to federal court, if P could have gone there in the first place. (This is consistent with Mottley.)o Can D remove on the basis of his “counter claim.”? – with P and D, it is sometimes a matter of who got there first. If D got there

first, he could have gone into federal ct. if he had a fed. claim.- On the face of P’s cause of action – fed. cause of action.

o D may bring later in another case if no intrinsic connection in federal court.P – state – DCC (fed. claim)- Can D remove? No, because he can only go on P’s counter claim. (P’s original claim should have been fed. one)

o Under Art.III Sec. I when §1331 was passed, it was written much more narrowly however, §1257 was written much broadly increasing/extending FQJ §1257 and others – constitutional (if there is some fed. q.) – even though they could not be brought under §1331.

o Congress can pass a § as long as there is somewhere a fed. issue. But, can’t pass a general §, that will be unconstitutional and will go against Art. III.

Moving back to GRABLE v. DARUE Falls between two cases: 1. Kansas City & Title 2. Merrel Dow v. Thompson

Smith v. Kansas City Title & Trust Co. – 1920 (391-394)Missouri bank invests in federal agricultural bonds. Shareholder’s complaint states that these bonds are illegal under the constitution, and under Missouri law you cannot invest in something that is unlawful. He sought to enjoin the bank from investing in these bonds.

Merrell Dow Pharmaceuticals Inc. v. Thompson – 1986 (394-401)Scottish and Canadian claimants sue D over Benedictin in part because it was improperly labeled by FDA standards. They have state law claims only – the FDA standard is not privately enforceable, but it is argued that it “creates a rebuttable presumption of negligence.”

1. Smith v. Kansas City & Title (1921): “Shareholders derivative suit”: is suing his own corporation bonds issue.- unconstitutional: They were federal bonds issued unconstitutional way by the corporation

Mottley: Fed. Q (one of the basis for well pleaded complaint) must be apparent on the face of a well pleaded complaint – it’s a cause of federal action.

o How is federal issue relevant in “Smith”? does it go beyond Mottley? – No federal clause of action (in P’s case).o In “Smith” – Outside of scope of Mottley. There is a federal issue under P’s case. – if a federal issue has to be determined, it doesn’t

require it to be P’s cause of action but has to be part of P’s case – could be a state claim depending on federal issue. o “Smith” enlarges FQJo “Smith” -- ∆’s case (entire action dependent on determination of federal issue.)o § or common law rights give P a right to recover.

2. Merrell Dow v. Thompson(1986)- P’s case, it’s a tort case (P’s trying to keep it in state court.)- P brought 6 causes of action, 5 – state law; 1 – loose interpretation (federal statute) – was also state law (but if interpreted § a particular way, an inference could be drawn with fed. law).

- Ct. – No Fed. Ct. JurisdictionIn Darue, “Smith” was distinguished from “Merrell Dow”. Darue, not governed by Merrell Dow but Smith.

o How did Smith go beyond Mottley?o How is Merrell Dow different from Smith?o Why is Darue more like Smith?

Mottley: Federal issue must be in claim itself.Smith: In order to determine a state action, must understand the (underlying) fed. q. first.

o Are bonds from fed. gov. illegal?o Similarly, in Grable – can’t answer until answer the fed. q.

In Merrell – Q of liability does not depend on FDA regulation issue. – still birth defects irrespectively (no matter the answer), hence, Tort Q. – Go to state – remedy depends on what state decides.

- Labelling doesn’t come into play.

C. SUPPLEMENTAL JURISDICTION

Supplemental jurisdiction refers to the various ways a federal court may hear either: state law claims, claims from parties who lack the amount in controversy requirement of diversity jurisdiction, when defendants are joined in claims, or when multiple plaintiffs are joined in claims, like in class action suits.- If state law claim arises from the same factual situation as fed claim not fair to make P go into both courts

1. PENDANT JURISDICTION2. ANCILLIARY JURISDICTION

4

Page 5: Exam Outline Civ Pro

PFQ Civ. Rule D1(state claim arising from same controversy)D2- Allows plaintiff to bring additional claims against additional parties or same d that would otherwise prevent them from getting into fed ct.- 1367a arise from same case/controversy part of the same facts applies to FQJ

- very broad supplemental jurisdiction- 1367b deals with diversitysupplemental jurisdiction not broad for diversity, as 1367a for FQ- Rule 14 – Impleader Situation ∆ can bring 3rd Party ∆ even if he does not meet diversity, still comes in. What doesn’t comes in are claims made by P against 3rd Party ∆, P X 3D (if jurisdictionally insufficient); PD3D: Each claim has to come as div/fq or supplemental jurisdiction.

- Owen Equipment v. Kroeger (deals with diversity): P can’t bring a non conforming claim against a 3rd party D. IF 3D not diverse it will be an end run around diversity D can sue 3D but P can’t (no end run around) - 1367b- Axxon/Ortega case : P1 (NY) suing for 100,000 ∆ (IL)

P2(NY) 10,000 same DCt says it can come in even if AIC not met If P2(IL) then, however, P2 cannot come in on a supplemental jurisdiction if no diversity but can come in if they fail to meet the AIC

Pendant Jurisdiction:1. Common nucleus of operative facts; 2. Substantial federal issue

Hence, Gibbs is bringing a suit under §303 which is a federal question does not permit violent action- Gibbs wants to bring pendant claim under State tort law – “intentional interference with advantageous commercial relations under Tennessee State law” (p975) But he cannot when Federal court listens to a case of State law by exercising pendant jurisdiction over the case that would otherwise not make it, it is very important for the federal court to be mindful of State court laws.- Federal law preempts the state law (ousts the state law) Federal law has precedence over State law.- Lots of State tort laws were preempted under §303 - but state tort remedies, violent activities survived.- Recovery of punitive damages and several other advantages not possible under Federal law, but can be claimed under state law.- Pendant State claim is brought in Federal court, Federal court has to apply State law but both cases will be heard, State and, Federal to which it is attached.- Federal court has to follow the interpretation of the law as laid out by the highest court of the State- Federal questions or diversity claims that anchors you into Federal courts is known as the anchor claim- Conditions for bringing in a Pendant Claim must arise from, 1. common nucleus off operative facts and, 2. substantial federal issue.

No diversity jurisdiction because he is suing a union, and they have citizenship everywhere for diversity purposes. This does not fall under §1331 because it is not a question of whether there is any federal claim at all, it is whether there is also

jurisdiction over the state claim. The existence of pendant jurisdiction does not depend on the success of the federal claim, it just depends on the existence of a

federal claim.

Anciliary Jurisdiction (Pendant Party)- The plaintiff and defendant have raised a standard case that ends up in federal court, but another party is joined that P may not be able to bring a case against in federal court.

- Mrs. Kroger, suing under state tort claim (wrongful death) diversity suit because a tort law.

UNITED MINE WORKERS v. GIBBSFacts: Gibbs was a manager who got in trouble with a union and was shut out of work. He sued the union for sort of conspiring to keep him out of the mining business. He had two theories and thus two claims: one state claim and one federal claim. The state claim was basically a tort claim. The federal claim was based on a federal labor statute.- He claimed violations of § 303 of the Labor Management Relations Act and a tort claim under Tennessee common law. Rule of LawA federal court can exercise pendent jurisdiction over state and federal claims if the federal and state claims are the type that would be expected to be heard at a single hearing and are “derive[d] from a common nucleus of operative fact.”-Whenever a federal claim that federal courts could properly hear under the “federal question” branch of subject matter jurisdiction is brought along with a state claim that comes out of the same facts as the federal claim, a federal court may (not must) exercise pendant jurisdiction.Issue: May a federal court hear a case based on a state claim if the federal claim is found to be wanting but the claims arise from the same factual bases?- Was it proper for the district court to exercise jurisdiction over the claim based on state law?Hold: The Court said that the district court acted properly in hearing the state case because there was a federal “pre-emption” issue.

Owen Equipment & Erection Co. v. Kroger – 1978 (423-429)<can’t violate congressional intent in pendant party jurisdiction>

Construction worker walks near a crane and gets electrocuted. Widow (Iowa) sues electric company (Nebraska) in federal court under diversity. Electric company claims it didn’t do anything wrong and brings in crane company under Rule 14. Widow files amended complaint, leaving out electric company and just naming the crane company (Nebraska). On the third day of trial (which is going poorly for the crane company), they mention that they are actually from Ohio. No diversity.

5

Page 6: Exam Outline Civ Pro

P(Iowa) OPPD (Nebraska) & above $75, 000 = Diversity- Under Rule 14, OPPD impleaded Owen this permits the ∆ to implead/brings in a 3rd Party ∆ who is Own; ∆ becomes 3rd Party P in their lawsuit against Owen Owen turns out to be Iowa resident (same as P.)- Couldn’t have sued Owen originally in Fed. Ct. as no diversity. - P sues Owen to get around diversity- ∆ Omaha is 3rd Party P suing 3rd Party ∆, Owen, but < $75,000 (3rd Part P also has to meet Diversity Jurisdiction) if $75,000 requirement not met, then you can come in Fed. Ct. as Ancillary Jurisdiction.-If federal question or diversity not met, then you can come to federal court under Anciliary Jurisdiction.- Here, two separate chain of casesFirst: 1. Kroger Omaha (original)

2. Omaha PleaderOwenSecond: Then, Kroger brings III party claim against OwenHence, judgment reversed. To permit Kroger to sue 3rd Party ∆ would be to permit a sneaky end run around diversity, hence, will not permit suit whether or not from common nucleus.- §1367 tracks the reasoning of Owen and Aldinger.- They both involve claims against other parties, that would not come independently different partydifferent claims

Aldinger v. Howard – (430-433)<can’t violate congressional intent in pendant jurisdiction>P got beat up by police officer, D. P says that D violated his 4th and 5th amendment rights. §1983 allows a person to sue someone who has violated their civil rights. Entities, however, cannot be sued under § 1983.Claim 1: P v. D for violating § 1983Claim 2: P v. City Police Dept. for state battery claim

This cannot be in federal court, because § 1983 specifically states that entities cannot be sued.o violating the purpose of the statute of not suing entities under § 1983 in federal court.

This is different from Kroger because this is not pendant party, it is pendant claim jurisdiction. § 1367(a) codifies this

Different from Gibbs as Gibbs had a federal claim and pendant claim one ∆, but in Aldinger, 2∆, bringing another party not originally part of the anchor claim. different claimdifferent ∆ anciliary jurisdictionAldinger said she can sue official under §1983, but SCOTUS wont permit supplemental jurisdiction to sue county (separate claim) – Lower courts permitted this on the basis of Gibbs test (but this was different from Gibbs).GIBBS 1367(a) “tracks gibbs” except §1367(b) “limits gibbs” – applies to §1332 (diversity)- Aldinger, Kroger, Finley overruled by 1367

Allapattah- Class action case lots of Pone meets amount in controversy in diversity but not others.- harder to meet complete diversity as number of P increases

STARKIST V. ORTEGAP1 (Ortega) – meets AIC ∆ Starkist (tort claim)P2 (Family)∆ Starkist, II tort claim of emotional stress, AIC not metDISSENT- can they get in on II claim? NO Relatives don’t meet AIC hence, both requirements of diversity not met, cannot come in In diversity case, you have to look at all the claims, even anchor to decideMajority As long as geographical diversity met, P2 can get in court meeting without meeting AIC permitted on basis of §1367(b)?Joinder under RULE 20 (permissive joinder)

D. REMOVAL

(p1000)§1441, §1446, §1447

P CANNOT REMOVE, only D can remove, but only on the basis of whether P would have gotten into fed ct. in the first place - Same principle

Owen Equipment & Erection Co. v. Kroger – 1978 (423-429)<can’t violate congressional intent in pendant party jurisdiction>

Construction worker walks near a crane and gets electrocuted. Widow (Iowa) sues electric company (Nebraska) in federal court under diversity. Electric company claims it didn’t do anything wrong and brings in crane company under Rule 14. Widow files amended complaint, leaving out electric company and just naming the crane company (Nebraska). On the third day of trial (which is going poorly for the crane company), they mention that they are actually from Ohio. No diversity.

REVIEW:Gibbs : Federal Question if the anchor claim is a federal question claim and there are other claims stemming from common nucleus all those claims can go in (even if under state claim) as all come out of state claim.Continuing with supplemental jurisdiction §1367 - after §1367 passage, if in federal court with anchor claim (state claim) §1367(a)[giveth] under Article III state claim arising from common nucleus of operative facts, federal court will take it.- §1367(a) is a very broad grant of supplemental jurisdiction-§1367(b)[taketh away] deals with §1332 (diversity cases) – in situations where anchor claim is diversity claim.Hypo: P(IN) ∆(IL) + state claim under §1367(b) same P, same ∆ hence, both come in under Div. in Fed Q but is P wants to sue II ∆; P(IN) D1(IL)+D2(IN) P cannot sue ∆ from same state.- §1367(b) under RULE 20 Joinder, you cannot join D1 and D2 as that would violate diversity and §1367(b) does not permit that.- But if P wants to stay in state court, he will bring in ∆2 to beat diversity.

6

Page 7: Exam Outline Civ Pro

as Mottley (well pleaded complaint rule)§1441(b)(2): if any of the D reside in the state where the suit is taking place, D cannot remove to fed ct As D can expect a fair treatment from his own state if 1 D from Ca and 1 in OH, same principle cannot remove, all Ds will have to agree to the removal

§1446: D has to remove within 30 days of getting the complaint§1446 also says that in no case it shall be more than a year or later. In case of diversity. No more than a year later unless a bad faith by P.

- Overlapping circles A:State Ct.; C: Fed. Ct. – overlapped area B – Removal involves cases where P could have gone to federal or state court (Mottley ruling applies to removal and P going to federal court in the first place)- Why P would choose to stay in state court? Fed Courts are overwhelmed (p1001) – federal judges are leaning on P and D to settle. Ds know that, hence, they try for removal.- Removal is mostly in Ds interest- “Forum Non-Convenience” (Forum non conveniens is a discretionary power that allows courts to dismiss a case where another court, or forum, is much better suited to hear the case. This dismissal does not prevent a plaintiff from refiling his or her case in the more appropriate forum.): easy to get case dismissed by D.P 1011-1012: New things happen as litigation unfolds – 1. Removal, 2. When can it occur, 3. Timing, 4. 144 (b), 5. Parties claims – added/dropped: state ct. claim may become fed. ct. claim and vice-versa.- What a P can do to defeat removal?

o In smaller amount case, can request < $75, 000o In big amount case, P can sue another D2 (bring in someone from state) and defeat diversityo Suppose D removes from state ct. to IN fed. court – D has 30 days, from the day removal info is received.

o For Div cases, 1year limit to remove (from filing of complaint)o FQ – you can remove even after a year

HYPO: p IN $1M d OH- P should go to OH, to remain in state Ct. and D can’t remove to fed. ct. as no prejudice issue. - 1441(b) – one way to prevent removal in div. cases (motivated by fear of prejudice) – go to D state court to beat prejudice issue.p.1011-1012: Joinder and Remand - 1441© permits D to remove an entire state court proceeding if it includes both a federal question claim under section 1331 and a claim not within the original, supplemental or removal jurisdiction of dist.ct. Upon removal, section 1441© directs that the dist. Ct. shall sever-and-remand the claims that do no fall within its original jurisdiction.- 1447© permits P to move to remand a case that was not properly removed.

A. THE TRADITIONAL FORMULATION: THE “POWER” THEORY OF JURISDICTIONPennoyer, Harris, Hess

Personal Jurisdiction – “where” question. Courts can only hear cases when they have PJ over the defendant. PJ=court’s power to hear case over a D.In personam jurisdiction: Jurisdiction over full individual : i) consent of the ∆; OR ii) serving the ∆ in forum state

IN PERSONAM- Jurisdiction against a person- Court will have access/control over all of the assets of a person if under

IN REM- Jurisdiction over a physical thing/over the property- classic proceeding is that of a quiet title (A proceeding to

QUASI IN REM- Not a quiet title form of action.- Jurisdiction – can be any claim but property is being

Part II(PERSONAL JURISDICTION)

CHOOSING THE FORUM – GEOGRAPHICAL LOCATION

Pennoyer v. Neff: Quasi In Rem Jurisdiction: Breach of contract for legal fees but using property to pull Neff in.-- No longer good lawEstablishes a very rigid rule, establishing PJ as part of what’s required under the DPC. It says that in determine PJ, look at presence in particular state and owning property in state allows court to exercise PJ in state. But Neff buys property after judgment and it wasn’t about property so court holds for Neff. Case says if in state, PJ, if out of state, no PJ. Quasi in Rem failed here.Neff, a non-resident, was summoned constructively (publication in newspaper) – SCOTUS found due process violated.1. In an action for money or damages, where ∆ is a non resident AND does not make an appearance, The court has jurisdiction only over the property which was attached before the lawsuit. 2. the law assumes that property is always in position of its owner therefore it’s seizure will inform him.3. constructive service is ineffectual for any purpose.Rule of LawUnder the Due Process Clause, no person is subject to the jurisdiction of a court unless she voluntarily appears in the court, is found within the state, resides in the state, or has property in the state that the court has attached (before the litigation process).

7

Page 8: Exam Outline Civ Pro

court’s jurisdiction. establish an individual's right to ownership of real property against one or more adverse claimants.)

used to have jurisdiction over the person to the extent of value and property.

HYPO: P in CA, suing NY D in CA State court.- You have to serve NY D in CA- Can’t serve out of state – CA doesn’t have power of jurisdiction, only notice is not enough; need both, power and notice.- CA state jurisdiction can only call NY D when served in CA State, this notice is CA State is what gives CA court power over the NY DNotice – personal service required; in state – hand it to them in some wayConsent to personal jurisdiction.  A defendant who has not been personally served in the jurisdiction can nevertheless voluntarily appear and submit himself to jurisdiction.  In such cases defendant is said to have "consented" to jurisdiction. 

In Pennoyer: Constructive service via publication in local paper – technical error- Irony of case is that ∆ never got notice; but real problem was that state didn’t have power over ∆- In conclusion: ∆ breached the contract therefore property was used to get jurisdiction over him however, since he didn’t won

property until after the case therefore Oregon court never had any jurisdiction over him – hence, no quasi in REM jurisdiction.HYPO: What if ∆ held property before case? Then, quasi in REM jurisdiction would have come into play

Type of service for each kind of jurisdiction: 1. In personam – Personal service2. In REM and quasi in REM (D is pulled through property) – Notice can be published but not important as it involves property and everyone basically knows that every person gets some kind of notice to property

Doctrine of Fraudulent Inducement : P cannot be pulled into state to be served through fraud.

In quasi in REM, if you attach property, the court can only award = value of property but no more, as court has jurisdiction power only over property

HYPO: NY D -- $10M suit, in CA court; he can only be sued up to the value of the property owned in CA. For rest of the money, P can sue D in some other jurisdiction for the remaining amount. – property is irrelevant to the substance of the case, only used to attach to the D to pull him in CA court.

HYPOS: P’s lawyer IN, doesn’t get paid for legal work by IN D; what if D leaves IN and moves to FL?- What if D has property in IN? P can get value = value of property only- Can D be sued in FL under “NEFF”, if no property – quasi in REM case? They can be sued in FL court but not a very attractive

option.- What if P doesn’t want to go to FL? - What about KY, where D has property? D can be sued quasi in REM in KY.- Kids in IN, D in FL, no property in FL? If they are back for some reason, can be served; but this could be a problem too.- Can you make “fraudulent inducement” to call them – NO?

HYPO2: Suppose your client gets struck by a FL motorist and he goes back to FL, what can you do?

Harris v BalkHarris (NC) owes Balk (NC) $180. Balk owes Epstein (MD) $300. Harris takes a trip to MD and Epstein serves him for the $180 he owes to Balk. Epstein names Balk as defendant, and grabs Harris as the $180 in debt. Harris pays the debt to Epstein, goes back to NC and Harris sues Balk for his $180. The case turns on whether MD had jurisdiction over Balk to adjudicate.

This is quasi in rem action. o Epstein attaches the debt owed to Harris as property Esptein exercising Quasi in Rem by attaching Harris’s debt to balk o The debt is the property.

“The obligation of the debtor to pay his debt clings to and accompanies him wherever he goes.” This case was an extreme consequence of Pennoyer consistent with Pennoyer if debt can be tangible Remember Balkanized Approach!

HESS v. PAWLOSKIDoctrine of consent constitutional- Two ways to exercise jurisdiction: 1. Presence; 2. Consent (in this case) – by serving REGISTRAR, “Sec. of State” – link between POWER AND NOTICE.

∆s from Pennsylvania heading up to Massachusetts, where they have a car wreck. In order for the person who got hit in Massachusetts to sue, they must serve the person that hit them personally in Massachusetts Rule (passed because of this) MA statute stating that a non-resident that enters and utilizes MA road is implicitly consenting to personal jurisdiction for suits arising from his motor-related activities.

8

Page 9: Exam Outline Civ Pro

HYPO3: D FL gets into brawl with IN P, will doctrine of consent involve? NO- D FL leaves, but has to be served in person, agent (consent) cannot be served here (under NEFF) – go to FL to sue or wait for him to come back Agent cannot be served as no consent of DHYPO4: Budweiser Corporate in MO – sells beer everywhere; sells to IN res. – becomes sick – can be W be sued?- 2 steps law involvement – corporation1. Consent2. If you do enough business = you are “present”; statute evolved to perform service of req. to REGISTRARHYPO5: P FL can’t sue D IN, cannot sue under Pennoyer if no business in IN at all.

B. THE SHIFT TO MINIMUM CONTACTSMinimum Contacts Test = International Shoe + Burger Kings (refer to review sheet)

International Shoe, McGee

WA claims jurisdiction on two theorieso “continuous and systematic” relations with the stateo activities within a state can be adjudicated if the action “arises out of” that activity in the state.

Court says D must have “minimum contacts” such that the suit “does not offend traditional notions of fair play and substantial justice.”

Due process depends on the “quality and nature of the activity” with respect to the claim against it. Corporation benefits from the laws of a state and this may give rise to obligations. In this case, D had continuous and systematic business, enjoyed the benefits of the state’s laws, had an impact on the economy, and

suit arose out of those activities. Therefore, it does not offend the “traditional conception of fair play and substantial justice.”

- 1945 case, an essence of a modern era case big shift in philosophy from Pennoyer, though not totally away.- Corporation considered a person under law- WA suing in WA; SCOTUS There is jurisdiction Indicated a shit in term Personal Jurisdiction over a ∆ in a state by the state can be had if “Minimum Contacts”- Traditional notions of Fair Play and Substantial Justice - Two consequences of this case: 1. Minimum Contacts 2. Don’t have to be in state to be served crucial links between power and service are broken new prevailing law therefore, after this in all previous hypos, ∆s can be served.This is a case of general personal jurisdiction A nexis between basis for jurisdiction and suit subject of suit can be different from basis of jurisdiction.

The minimum contacts test: International shoe the courts of a state may exercise personal jurisdiction over defendant if she has such minimum contacts with the state that it would be fair to require her to return and defender lawsuit in that state.- whether jurisdiction is permissible depends on the quantity and nature, even a single contact would do, but not casual or isolated.- a Corporation that uses to conduct activities within the state accept reciprocal duty to answer for it’s in-state activities in the local courts. the defendant to deliberately chooses to take advantage of the benefits and protections of the laws of a state will not be heard to cry “foul” when that state holds her to account in its courts for her in-state acts. voluntary relation between the defendant the state power to exercise jurisdiction arising from that relation.- therefore, under international shoe, the corporation could not have been required to defend a claim in Washington arising from shoe sales in Texas under the minimum context analysis, it must always consider the relationship between the contacts that give rise to the suit and the state where the suit is brought.

- If Minimum Contacts In personam jurisdiction can be attainedAfter International Shoe In Personam is about minimum contacts

Cases where defendant has no contact with the forum state, State has no authority exercise personal jurisdiction over the defendant, unless she consents to it.

International ShoeFacts: DE Corp. based in MO does business in WA through independent contractor salespeople. Washington is trying to get the company to ante up for its unemployment fund. Washington served Shoe notice of assessment by personally delivering it to one of their salesmen in Washington as well as sending registered mail to their home office in St. Louis.Procedural History: Shoe made a special appearance in Washington to argue that the service of process on the salesman was not proper and that Shoe wasn’t doing business in Washington according to the statute. The courts in Washington found that Shoe was amenable to personal jurisdiction in that state. Shoe appealed up to the U.S. Supreme Court.Issue: Under what conditions is a corporation subject to personal jurisdiction in a particular state?Rule: A corporation that is protected by the laws of a state shall be subject to personal jurisdiction in that state.Analysis: The court interprets the due process clause and the Fourteenth Amendment to mean that if a company has “sufficient contacts” in a state, they may be subject to being sued in that state.

9

Page 10: Exam Outline Civ Pro

SMJ + PJ both have to be satisfied in federal court

McGee v. International Ins.Co. jurisdiction arising out of Single act specific in personam jurisdiction upholding jurisdiction over a claim arising out of a single contract solicited in the state.- High watermark of minimum contacts most expansive view of minimum contacts- CA P. TX ∆ , can pull, the one contact is enough CA hold PJ the contact must happen in said state.

PURPOSEFUL AVAILMENTpurposefully availing itself of the privilege of conducting activities in the forum state, availing the benefits and protections of its laws. defendant must have made a deliberate choice to relate to the state in some meaningful way before she can be made to bear the burden of defending there. this case is different from McGee

defendant must have purposefully availed itself of the benefits and protections of the forum state’s laws to have minimum contacts Did

∆ took advantage of state in anyway, some attempt by ∆ to direct his act to that state. Purposeful Availment is a limiting principle.

C LONG ARM STATUTES

Moving to Purposeful Availment, Stream of Commerce, Long Arm StatutesStatutory limits on personal jurisdiction

Due process clause of the 14 th amendment imposes fundamental limitations on the power of state courts to exercise personal jurisdiction over defendant in civil suits. States may only assert jurisdiction over defendants who have established a significant relationship to the forum state, such as domicile, in state presence, continuous and substantial business within the state, consent to suit in that state, our minimum contacts with the state that gave rise to the claim in suit. only defines the outer bounds of permissible jurisdictional power.

Two-step analysis first, whether there is a state statute that authorizes it to exercise personal jurisdiction under the circumstances of the case, second, if there is, whether it would be constitutional under the due process clause to do so.

Long arm statutes authorize courts to exercise jurisdiction over defendants based on specific types of contact with the forum state. International shoe and succeeding cases States adopted long-term statutes to authorize their courts to hear cases arising out of such contacts. for example cases such as transacting business in the state, owning property, committing tortious act within the state etc. a good deal of jurisdiction authorized by the due process clause but not all of it.Primary purpose of the long-term statutes to reach out that the state to call nonresident defendants back into the state to defend lawsuits; even though the defendant has left the form state before he he is sued, he may be required to defend a suit there under the international shoe analysis if this suit arises out of his prior contacts with the forum.

McGee v. International Life Ins. Con.Pennoyer Hess Int’l Shoe McGee

Californian resident buys a life insurance policy with Texas co. That Texas Co. gets bought out by an Arizona co. and the Arizona co. voluntarily takes over the policy. There is a dispute over the policy and P sues in California. (California has a long-arm statute specifically naming insurance companies.)

Rule: If the contact between the defendant and the state of California is minimal, such contact must be closely related to the claim in order for the state of California to have jurisdiction One contract is enough for the consumer to get the business.

Reasoning Specific jurisdiction is being asserted here Using the Int’l Shoe “minimum contacts” does not offend “fair play and substantial justice”. California has a manifest interest in the insurance claims of its citizens burden on P would be great if Californians had to go elsewhere

Hansen v. DencklaFacts: A family fought over the estate of Mrs. Donner, who had established a trust in Delaware but had then moved to Florida and died there. If Florida had jurisdiction, one daughter would get everything, while if Delaware had jurisdiction, three daughters would share. Issue: Are there sufficient contacts between the trust company and the state of Florida for the state to have specific jurisdiction? Rule: If there are only minimal contacts between a state and a defendant, then the contacts must be closely related to the claim. Analysis: The court says that the contacts were not sufficient in volume, nor were they sufficiently related. The court concedes that the trust paid income to her in Florida and she did a little bit of work on administering the trust from Florida, but the court says that’s not enough. Conclusion: The court rules that Florida does not have specific jurisdiction over this matter.

10

Page 11: Exam Outline Civ Pro

should not exceed its constitutional grasp refuse to apply to cases that fall outside the bounds of due process all long-term statutes that base personal jurisdiction on specific enumerated acts require that the claim sued upon arise out of The act itself Limited in state contacts only support jurisdiction over claims that arise from those contacts.

Stream of Commerce: Gray; Volkswagen; Kulko, Asahi, J. McIntyre

D REFINING THE MINIMUM CONTACTS ANALYSIS

World-Wide Volkswagon Corp. v. Woodson – 1980 (131-140) [Sept. 14]

SMJ + PJ may sometime clash with one another.- P is a domicile of NY as hasn’t reached AZ yet. Have yet to establish residency.- Domicile is not necessary in PJ- This case, P suing in OK St. Ct. as juries pro P, hence, P want to stay there hence, keep out of federal court in OK, otherwise will be transferred to NY (jury not pro P)- OK long arm § State court says OK has jurisdiction OK superior court says if NY (Seaway and VW) derives substantial revenue from goods used/consumed in the state This is the only car entering the state that “we” know of.- Statutory language is the basis for court’s jurisdiction and OK is trying to stretch it very loose reasoning.

Calder v. Jones – Purposeful Availment + Minimal Contacts stands

P. + Publication both, very strong case

- It’s an intentional torts case, combination of tort + constitutional law punitive damages can be recovered with intentional tort but not for negligence tort, hence, Jones needs to show that Clader and Party were lying, it was an intentional tort.

the defendant publishing allegedly defamatory article about the California actress I’m distributing it in California held as a deliberate reaching into California.

Gray v. American Radiator & Standard Sanitary Corp. – 1961 (118 – 128) – first SOC case very influential casePennoyer Hess Int’l Shoe GrayTortGray builds a valve in Ohio, it is put in a boiler in Pennsylvania and then shipped to Illinois where it blows up.

Reasonable inference that Gray’s product would lead to “substantial use and consumption” in Illinois. Gray enjoys benefits from Illinois’ law and protection. it was foreseeable that the product would be in Illinois, regardless of middlemen. Modern times make commercial activity in other states necessary and foreseeable, not inconvenient for D to

defend there because of transportation and technology. long-arm statute

o the long-arm says that “the commission of a tort within the state creates long-arm jurisdiction”- IL says, well settled that a tort occurs where final act occurs

- ∆ says no Purposeful Availment as OH ->PA but no contact with IL- Hence, IL Supreme Court broadly interprets statute to develop “SOC”Court concluded that The Ohio water maker the tendon nothing in Illinois had committed atrocious fact in the state, since explosion that injured the plaintiff to place in Illinois. In this case even though the defendant did not act in Illinois, did not send good stair, and did not know I anticipate that its products would end up in Illinois, still IL long arm statute was exercise over him.

Pennoyer Hess Int’l Shoe Gray WW VWTortRule of Law: Foreseeability alone is not sufficient to authorize a state court’s assertion of personal jurisdiction over a non-resident defendant that has no contacts, ties, or relations with the forum state.Facts: NY resident drove the Audi she bought in NY through Oklahoma where it blew up. She is suing the manufacturers and distributors, and wants the NY dealership (WW VW) in the suit so they can’t claim full diversity and move the action to the federal court. WW VW sells exclusively in the tri-state area.

In this case, they don’t avail themselves of Oklahoma.o If they had sold many products to Oklahoma residents, then it would be fairer.

“’foreseeability’ alone has never been a sufficient benchmark for personal jurisdiction under the Due Process Clause.” law should “give a degree predictability to the legal system that allows potential defendants to structure their primary conduct

with some minimum assurance as to where that conduct will and will not render them liable to suit.” unilateral action of the consumer to bring a product into a state does not give jurisdiction by itself.

SCOTUS (P.Hill’s notes): Too broad to take on personal jurisdiction; purposeful availment – not met necessarily.Why not Gray’s Reasoning? Because SOC ends where P buys the vehicle at the point of sale.

11

Page 12: Exam Outline Civ Pro

- defendant may have sufficient contacts for the state to support minimum contacts just diction there even though she did not act within the state. If a defendant coming select outside the state that she knows her cause harmful effects of it in the state, she may be subject to minimum context jurisdiction there for claims arising out of that Act

CALDER EFFECTS: 1) committing an intentional act, 2) expressly aimed at forum state, 3) causing harm, the brunt of which is suffered

in forum state

Hypo – Bomb TX CA (Explodes in NM enroute) yes, can sue in NM (Like shooting an arrow and somebody else coming in between –

recklessness) – Where activity is directed, where P lived, where it occurred?

Baltimore Case (note # 6) – Trademark infringement (just because everything is happening in Baltimore, doesn’t mean that it is not effecting IN)

Hypo: Can IU McKinney law student sue law professor for writing harmful LOR for job in CA NO, as some effects will aim at student and

law firm in CA, however, all activities centered in IN.

Keeton v. Hustler the defendant to purposefully availed itself to the opportunity to engage in In-state activities, by distributing its magazines with interstate Pitch those contacts supported jurisdiction even though the defendants act had greater impact in other states, and the plaintiff a few contacts with the forum state.Rule: Personal jurisdiction is proper over a nonresident magazine in any state where that corporation has sold and distributed a substantial number of copies.Hustler purposefully availed itself of the privileges of doing business in New Hampshire, and should reasonably anticipated being haled into court for claims related to the magazine it sells there. National publications may properly be sued for their content anywhere “a substantial number of copies are regularly sold and distributed.” The ruling of the court of appeals is reversed, and the case is remanded.

Kulko v. Superior Court of California – 1978 (142-144) [Sept. 20]Stream of CommerceDivorced couple, father lives in NY, mother lives in Cal. The kids live with the father but are unhappy, so he sends them to Cal to be with his mother. Mother sues father for child support in Cal.

Supreme Court doesn’t want to give jurisdiction to Cal., because it creates the wrong incentives. The father did a good thing, and it would punish him if they let the mother adjudicate in California.

father did not purposefully avail himself of CA laws by sending daughter to livethere o Did send child support checks there, but that was court-mandated (not voluntary)o Policy reasons of not wanting to discourage divorced parents from moving for fear of jurisdiction

Burger King v. RudzewiczBurger King v. Rudzewicz – 1985 (161-173) [Sept. 21]Pennoyer Int’l Shoe Hess McGee Burger KingContractFranchise buyers in Michigan get in a dispute with head office of BK. BK sues them for contract and tort damages ∆s, subject of a suit in FL even thought they never actually went to FL.

There is a two-part analysis:o minimum contacts purposeful availment + stream of commerce

90% of the time, most of the actiono Fairness and Efficiency (Minimum Contacts has to be consistent with notion of fair play and substantial justice.) – is it too

harsh on ∆, does FL has a personal interest does it makes sense to force ∆ to come to FL.Bulk of the contract goes to step 1Both these parts are not entirely independent. - Expansive construction of PJ (opinion written by Brennan)- we look at the amount of burden on ∆; forum state’s interest in adjudicating.- burden on P to establish min. contacts by one factor first move to other factors ∆ has higher burden to show.- looking at the contract (midway between long arm relationship with burger king and this franchise issue that came after) long term relationship MI did derive benefits- Center of gravity FL Bound by FL law in contract (P 815 – Choice of Law Analysis) COL gives additional reason to make FL the appropriate place. COL points to FL, hence, appropriate to go there.- inconvenience comes in here jurisdiction that would render litigation “so gravely difficult and inconvenient that a party will for all practical purposes be derived of his day in court.”- However, with modern technology and ease in travel, going to another forum is not that difficult.- Burger king: Specific personal jurisdiction supported where continuous but limited activity in the forum state, such as ongoing business relationship. the defendant has purposefully dedicate activities to the forum state, jurisdiction can be reasonable.

NICASTRO (RECONSIDERING SOC THEORY AFTER ASAHI)

In Nicastro, a four-justice plurality held that a defendant must target a forum with its products in order to be subject to personal jurisdiction there for lawsuits relating to its products under a stream of commerce theory, and the mere fact that a defendant may have predicted that its products would wind up in the forum is not enough to subject the defendant to personal jurisdiction for such lawsuits. Nicastro, 131 S. Ct. at 2789-90.

12

Page 13: Exam Outline Civ Pro

The plurality held, thus, that the defendant in Nicastro was not subject to personal jurisdiction under a stream of commerce theory, as it had not targeted the forum with its products. Id. A two-justice concurrence agreed that the defendant in Nicastro was not subject to personal jurisdiction because the case at bar involved only one isolated sale of a product, which could not give rise to personal jurisdiction under any circumstances.

Nicastro v. J. McIntyre Reconsidering stream of commerce – J. McIntyre Machinery: defendant made a metal shearing machine in England and sold it to distributor in the United States which resulted in New Jersey. The plaintiff was injured using it in New Jersey and sued there for his injuries. Majority held that Contacts in New Jersey would not support the Specific in personam jurisdiction in New Jersey for nicastro’s claims. even though stronger than Asahi, as defendant had reached out to sell his products in the United States, caused injury though indirectly in NJ not only knew that his products were entering the US market but promoted that market. argument-purposefully availing to the United States but not to the states where its products were resold. Majority opinion - jurisdiction may be proper if the defendant regularly serves in the market in the state where its product causes injury. Manufacturing will not be subject to jurisdiction in another state Data distributor reasons the goods.

Two points: first, foreign manufacture selling credits to wholesalers outside United States without cultivating the US market-no jurisdiction. Second, selling goods and encouraging sales, jurisdiction in the state fair dedicates good, that does a US distributor is located.

In conclusion purposeful availment must establish first after that other factors in determining whether the exercise of jurisdiction would comport with “fair play and substantial justice”.

Hill’s Notes: Men(plurality) – No Jurisdiction; Women (Dissenting – all 3 women) – JurisdictionPlurality – its not the expectation/foreseeability but the action- plurality talks about jurisdiction being everywhere in that case needs some sense of connection with the state.

Asahia manufacturer sells finished products to wholesaler outside the state, the wholesaler then resells to retailer in the forum state, and the retailer resells to the consumer the party in the beginning of the stream of commerce did not import the production of the forum state itself, it sold to others who did The manufacturer or component maker may or may not know or care about the Ultimate destination of product split court in Asahi. O’Connor “mere awareness” of product reaching the forum state through stream of commerce, not enough for “purposeful availment”. Clear evidence required that the defendant seeks to serve the market in the particular state, suggest designing the product for the market in that state or advertising there. Majority view: sending goods into the stream of commerce at least in substantial quantities constitutes purposeful availment, even if original maker unaware or goods being sold there or cultivating customers.- even if minimum contacts established, unreasonable to exercise jurisdiction on the unusual facts of the case.

Asahi Metal Indus. Co. v. Superior Court of California – 1987 (145-154) [Sept. 20]Pennoyer Int’l Shoe Hess Grey AsahiTortTire valve built by Asahi (Japanese corp.) was faulty causing an accident in California. Asahi sold their product to other corp. that sold to California. Their product was foreseaably going to the California market.

International companyStream of Commerce

4 justices say no, 4 say yes, 1 says (in dicta) enough quantity the 4 no’s – O’Connor

o quoting WW VW, foreseeable is not enough. Instead, the action must be purposefully directed toward the forum State. – Stream of Commerce PLUS

o additional conduct from P, instead of just putting it into the market: designing it for the forum state, advertising in forum state, providing advice to members of the forum state.

the 4 yes’s – Brennano don’t agree with the purposefully directed statemento ‘stream of commerce’ refers to the “regular and predictable flow” of a producto manufacturers availment – reciprocity

benefits from the state’s laws indirectly benefits economically from the state itself defendant accrues benefits from that stream regardless of direction

Pavlovich 4-3 ruling CA did not have PJ Calder’s three part test was used 1) committing an intentional (tort) act, 2) expressly aimed at forum state, 3) causing harm,

the brunt of which is suffered in forum state (effects) Was the ∆ aiming 2 factors – 1. Is website interactive? 2. Is there some commercial transaction? neither present in Pavlovich. Dissent Thinks there is jurisdiction licensing agency started working after two months, but he still knew about the movie and

computer industry. - this is a threshold issue Learn to make connection between this case and Calder, same goes for Keeton case as well. - Calder sent magazines physically into CA to addresses, to stores made money - Pavlovich didn’t make money

13

Page 14: Exam Outline Civ Pro

- Is Pavlovich doing the harm? Any harm occurring, occurred by third parties the VW principal applies here because its third parties.

EXAM: Put the facts up beside one another.

E PRESENCE OF DEFENDANT’S PROPERTY

Shaffer v. Heithner: SCOTUS will use min. contacts in in personam context like in int. shoe. It was first case since pennoyer that was quasi in rem. Reducing three categories to min. contacts. Even though in rem and quasi in rem contracts are being cont. used as labels but behind them lies min. contacts.There were 28 people out of which 21 got sued, who were stockholders and on that basis DE asserted jurisdiction. The other seven did not have stock hence, weren't sued.Justice Brennan’s dissent: P. is suing as the ∆s ignored court order. The irony/real reason is that under DE law he can only sue 21. It’s the ownership of the stock and not the things they did as officers that permitted quasi in rem jurisdiction. Brennan things there were min. contacts because of the activities they engaged in as officers if the corporation. He thinks P. should have been able to sue all 28. They all agree that min. con. will now be the standard but other do not agree that there were min. contacts.

Hypo: Suppose P. lives in CA and wants to sue ∆ in CA court. ∆ lives in NY, has no business contacts but a million $ home in San Fran. This is a $10 M. Suit.Under Pennoyer can the ∆ be sued in CA? Yes, under Quasi in Rem for $1 M. Jurisdiction= value of property.Under Shaffer? No Jurisdiction as no minimum contacts. Quasi in Rem J. goes away. P. will have to go to NY to sue him.

Hypo 2: Suppose the CA P. instead is renting his property of ∆, whose guest ruins it. Can he sue him now in CA? Yes he can, as the suit is related to property, now there is min. cont. He owns property and someone is injured on it, min. contacts. Suppose P. has a $10 M suit, but prop. is $1M, but because of min. contacts now it is in personam min. contacts and hence, P. can go after all $10 M. Court call is Quasi in Rem 1, in Shaffer. Jurisdiction is related directly to the property. Prop. is not just being used to pull the ∆ in.

Shaffer v. HeitnerRule of LawQuasi in rem jurisdiction may only be asserted when the interests of the persons in the property seized have sufficient contacts, ties, or relations to the state.

FactsHeitner (plaintiff) brought a shareholder derivative suit in Delaware against the Greyhound Corporation, incorporated in Delaware, represented by Shaffer, alleging acts that took place in Oregon. Heitner also filed a motion for an order of sequestration of shares of Greyhound stock owned by the individual defendants named in the complaint, who move to vacate the sequestration order on the grounds that the ex parte sequestration violated their due process rights and that the property seized was not capable of attachment in Delaware. The court rejected this challenge on the ground that the suit was brought as a quasi in rem proceeding, which is traditionally based on seizure of property present in the jurisdiction rather than contacts between the defendant and the state.IssueWhether there must be sufficient contacts between the state and the interests of the defendants in the property seized before a court can exercise jurisdiction quasi in rem.Holding and Reasoning (Marshall, J.)Yes. Quasi in rem jurisdiction may only be exercised over a person if the interests of the person in the things seized meet the minimum contacts standard established for personal jurisdiction in International Shoe Co. v. Washington, 326 U.S. 310 (1945), which established the minimum requirements for exercising jurisdiction in a manner consistent with the Due Process Clause of the Fourteenth Amendment. Thus, if there were not the minimum contacts required by International Shoe, the Delaware sequestration statute deprived Shaffer of his property without due process. Here, the property seized was statutorily located in Delaware, but it did not have any relation to the subject matter of the litigation. The defendants had nothing to do with the state of Delaware; their only connection to it was that they had accepted directorial positions in a company incorporated in Delaware, but accepting these positions does not mean they can be taken to a Delaware court. Accordingly, Delaware’s assertion of jurisdiction over the defendants was inappropriate and the judgment below is reversed.Concurrence (Stevens, J.)Purchasing stocks on the open market, where the purchaser may not know in what state the corporation is incorporated, is not a sufficient contact with the state to establish jurisdiction. However, the scope of the Court’s decision is uncertain and might be applied inappropriately in other contexts.Concurrence (Powell, J.)The International Shoe rationale should not necessarily extend to other forms of property, such as real property, located within the state.Concurrence/Dissent (Brennan, J.)The International Shoe standard should apply to the case. However, in accepting positions with a Delaware corporation, the defendants elected to assume powers and to undertake responsibilities wholly derived from Delaware’s rules and regulations, which amounted to sufficient contacts to justify exercise of jurisdiction.

14

Page 15: Exam Outline Civ Pro

F PERSONAL SERVICE WITHIN THE JURISDICTION

Burnham jurisdiction based on in-state service only requires the defendant to be present in the state at the time that the summons and complaint are served upon her may not have any contact with the state at the time of the events giving rise to suit.

This is Pennoyer style Tagging someone when they are in state. [In Kulko case, court disagrees. Court says that in future people won’t let children visit other parent. This case is in the background here.] No minimum contacts following Kulko.

Why no min. contacts, since he was in CA and was seeing his kids? He was there for a business trip unrelated to family law issue. Incidentally did see his kids, not enough to give CA court Jurisdiction, according to most members of court. Has he purposefully availed himself? Justice Brennan is concurring but does thinks there are min. contacts. Brennan tries to distinguish it from Kulko.

All agree with the decision, fighting is for the rationale. Scalia is a textualist : interpreting the due process clause. Thinks jurisdiction is fair on basis of Pennoyer.

Brennan says there are minimum contacts. Also, if you don't have min. contacts than jurisdiction is not appropriate with due process clause. What test should be used to determine the due process clause?

Test: Scalia for Textualist, Due Process, Ultimate conservative; Brennan for minimum contacts; Other two Judges who are moderates. On exam talk about all of these.

Scalia: Theory of Textualism, in this case Due Process Clause, its meaning has to comport with what most people associate its meaning with and not when it first came of in 1800s. According to textualist approach it’s illegitimate to take a clause that originally meant something else and apply it today For him the meaning of constitutional causes is locked in time. Judges should do best to historically understand the meaning when it was ratified.

Brennan it’s evolving according to changing social attitudes. We live in modern times, the court has to be practical. Court has the authority to

Burnham v. Superior Court (review) – 1990 (198-210) [Sept. 7, 23]

Guy has divorce dispute with his wife in NJ. She moves out to California, and he flies there on business, and also visits his kids. The wife serves him.Rule: A non-resident is properly served if he is physically present in the forum state, and the forum state may exercise personal jurisdiction over him without violating due process.

Husband is arguing that he has no International Shoe minimum contacts with California, and so under Shaffer v. Heitner he should not be allowed to be served.

Holding and Reasoning (Scalia, J.) -Yes. A non-resident party may be properly served with process while temporarily visiting a state without violating due process. It is well established that states have jurisdiction over non-residents who are physically present in the state, no matter how long that individual plans to stay in the state. This rule has a long and continuing tradition of being enforced by the courts, across jurisdictions. Burnham argues that he lacks the minimum contacts necessary for the court to properly apply personal jurisdiction, but that is a perversion of the standard; indeed, the minimum contacts standard was created for use in the absence of a party’s physical presence in the state, which is not necessary in this case, as Dennis was physically present in California when he was served. For the same reason, Dennis cannot rely on the holding in Shaffer v. Heitner, which held that a state lacks jurisdiction over an individual unless the lawsuit arises out of his activities in the state, because the non-resident litigant in that case was an absent one, unlike Dennis. Justice Brennan’s concurring opinion disregards the fact that this body of law is based in traditional notions of due process, not fleeting or contemporary notions. Because Dennis was physically present in California when he was served with process, the state properly asserted personal jurisdiction over him, and the holding below is affirmed.Concurrence (Brennan, J.) - A long history of enforcement of personal jurisdiction when a non-resident party is present in a state, while informative and persuasive, is not necessarily dispositive. It does mean, however, that defendants have notice that they might be served if they enter a different forum state. By entering the state, the defendant avails himself of the benefits made available by the state, such as police protection and use of the roads. Furthermore, the burden on the non-resident defendant is small; modern travel is not a great inconvenience for an individual who wishes to defend himself in a foreign state.

15

Page 16: Exam Outline Civ Pro

improvise. After this case we can serve someone in the state without minimum contacts but not fraudulent cases. Presence in the jurisdiction without min. contacts is going to be enough. Vestiges of Pennoyer continue to persist.

Theory of Acquiescence: If something has been accepted constitutionally for a long time, the court frequently says it’s hard to say how it is not fair as it goes back so many years and we are not ready to throw it out. This case is not about min. contacts but whether you are in the state.

G THE GENERAL JURISDICTION ALTERNATIVE: 1. GOODYEAR; 2. HELICOPTEROS

- This case is about General v. Specific Jurisdiction Did any tire got into NC through the stream of commerce? Not that type but others did.- Reminds of Grey v. American Radiators, how is this case different? In Grey did the valve that caused the accident went to Illinois? Yes they did.— What about the tires in this case? No, they did not wind up in NC, the ones that caused the accident. — Is there specific jurisdiction? No, general jurisdiction. Specific J. involves a claim of jurisdiction which is a nexus between an event that gives jurisdiction and the accident. There has to be a nexus between defendant’s contact with the state and the reason for the suit. —Specific Jurisdiction - When basis for jurisdiction is directly related to the suit itself. The nexus between state’s basis and what ∆ did In Burger King, the jurisdiction was specific as it was in regards the contract. There was a nexus, basis for which was the contract. —In Volkswagon no jurisdiction, would have been specific if suit would have taken place. — In Int. Shoe, specific jurisdiction Whenever we talk about min. contacts, the court has jurisdiction for purposes of that suit BUSINESS WAS THE BASIS— When particular business is in a particular state with systematic and continuous activity, general jurisdiction as the business can be sued for any unrelated reason as well. This was before this case.— Is systematic and continuous business enough for general jurisdiction? There will be specific jurisdiction. Not general jurisdiction as state has to be your home, you have to be located there.— General Jurisdiction: If you are a resident of a state, that state has general jurisdiction over you, even for things happening in other sate. Similarly, corporations in a state are like people in that state and can be sued for unrelated things.— Specific Jurisdiction: there has to be a connection, a minimum contact. ∆’s activity in the state have to be the basis for jurisdiction in that state.— General Jurisdiction: when specific jurisdiction fails but ∆ lives in the forum state, hence, can be sued there. ∆ has to reside there as an individual or as a business.

I THE REQUIREMENT OF NOTICE – p905

Notice and Manner of Service

Notice: governed by constitutional requirements. Whether P. did all that he reasonably could and ∆ got notice of the case. Governed by the due process clause. Generally, a P. should do everything he reasonably can to assure that the affected party gets notice of the hearing so that they can come and represent themselves.

Test: P. should do everything…. (as above) It’s a fairness standard. In Mullane, Court said publication is not enough personal notices should have been sent. MULLANE: Trust company aggregates small trusts into one big one. The co. protects itself from litigation by beneficiaries by getting declaratory judgments periodically saying that its mgmt. is ok. It serves people by mail and by newspaper ad. Not all beneficiaries are able to be located, and there is inherent tension between principal and interest beneficiaries.

GOODYEAR TIRES: Rule: A state court may not exercise general jurisdiction over a foreign subsidiary of a United States-based corporation unless it engages in continuous and systematic activities in the forum stateThe Supreme Court held that foreign subsidiaries of Goodyear were not subject to general personal jurisdiction in North Carolina, even though several thousands of their tires were distributed in North Carolina. The court rejected the argument that sales into the state suffice to support general jurisdiction. Corporation subject to general in personam jurisdiction in the state where its incorporated and its principal place of business Section 1332 C1 defines the corporation’s state citizenship for diversity purposes.

In Helicopteros Nacionales de Colombia, S.A. v. Hall, 466 U.S. 408 (1984), survivors of United States citizens who died in a helicopter crash in Peru filed wrongful-death actions in Texas state court against the owner and operator of the helicopter, a Colombian corporation. The Colombian corporation had no place of business in Texas and was not licensed to conduct business in the state. While the corporation made some purchases in Texas, these activities failed to achieve the required “continuous and systematic business contacts” noted in Perkins.

GOODYEAR again…Here, the sale of some tires in North Carolina through intermediaries is insufficient to warrant jurisdiction over them. Measured against Helicopteros and Perkins, North Carolina is not a forum in which it is permissible to subject the subsidiaries to general jurisdiction. The judgment of the North Carolina Court of Appeals is reversed.

16

Page 17: Exam Outline Civ Pro

If there is a reasonable way to contact the party do all that you reasonably can, don't do just the minimum requirements. There cases arise when there are multiple parties or the P. genuinely doesn't know where the ∆ is.

If you think ∆ is in state then you publish (when you don't know the exact address.) Publication is discouraged today, specially Mulane onwards.

If ∆ is out of state publication is not enoughIn Pennoyer type situation, today pub will be req. in same state where ∆ is and not other. Default judgment: when ∆ doesn't show up in court, P. gets a default judgment but there is no adversarial process. Default judgments

are notoriously week. And if ∆ hasn’t gotten notice he can come back later with notice and get the judgment vacated. If ∆ got notice and P. reasonably did everything he could then ∆ will have a hard time, otherwise DJ vacated. Notice by email is not sufficient unless the other person consents to it. The initial notice without consent is constitutionally insufficient. When ∆ doesn't get notice, there is not an opportunity to test the P. In multiple ∆s, even if one/two ∆s doesn't get notice, P. can still be tested and those ∆s cannot re-litigate.

Service of Process: Governed by state rules (rule 4)

Classic form: personal service, handing it in hand. Leaving summons at place of aboard if someone is of 18years of age(4e2) SOP is governed by local rules.

J VENUE

Venue is determined by § 1391, which gives the rules where a federal action may be brought (either under diversity or a federal question). Venue acts as a funnel, limiting the places where a case can be brought.

For diversity, § 1391 says that venue is had where:1. everybody resides if they reside in the same state, or2. a “substantial part” of the event occurred3. any defendant is subject to personal jurisdiction, if you can’t get them under the first two.

For an action not based totally on diversity, venue is had where:1. everybody resides if they all reside in the same state, or2. a “substantial part” of the event occurred.3. any defendant may be found., if you can’t get them under the first two.

Corporations ‘reside’ everywhere there is in personam jurisdiction over them.Individuals ‘reside’ in one state only. “may be found” means actual presence, not just amenable to jurisdiction.

The structure of the statutePart (b) deals with not-diversity, or “federal question”-type cases. Part (c) is a special provision for corporate defendants.

Hill’s Notes: § related to venue changed in 1990 and 2011.Why do we need venue? Why separate requirement? Why is it above personal jurisdiction? Venue: for court’s convenience, ∆’s too.Because PJ can be over a ∆ in more than one place as there are min. contacts over more than one place. Venue statute is 1391 (amended again 2011).1391 b: gives basic requirements of venue. b1: Relevant in any district in which a ∆ lives if all ∆s live in the same state.b2: contemplating the convenience the ∆,1391 b2: deals with the events of the case. Where does the substantial part of the events to. prop. etc is appropriate for venue.VENUE is district by district and not state by state.b3: fall back position when there is no venue and pj.Difference between diversity and fed question before the amendment but more similar now.1391c: where people are, where are they domiciled, same test as Mass v. Perry.Corporations, business under c too. Very loose provisions.

Hypo: P. sues two brothers (B1, B2) in breach of contract.P: IN wants to sue in INB1: INB2: ILCan P. bring the suit in a district in IN? You can bring suit in any district in which ∆ lives if all ∆s live in same state. For B2 bring suit under b(2)

Venue requirements apply in federal court and not the state court.

If P. buys a vehicle from ford in southern district of NY, has his breaks fixed in Western Dist. of NY. Accident occurs in NJ. Ford’s venue? look at c, it’s not limited. For venue principle place of business is not needed for corporation, personal jurisdiction is enough.

17

Page 18: Exam Outline Civ Pro

Wants to sue break shop and ford in southern district? Yes he can as all ∆s in NY, ford is subject to PJ.Can he sue in NJ? Yes, he can under b2, accident is substantial part of the event. Personal Jurisdiction: Venue enough.

There are certain timing issues If you don't claim PJ you waive the objection, C&S got into the venue thing.

After this case, talk about the language of b2The substantial portion language in b2 is pretty loose.

K DISCRETIONARY DECLINE OF JURISDICTION- Deals with Forum Non-Conveneins and Transfer of Venue

Forum non conveniens is a discretionary power that allows courts to dismiss a case where another court, or forum, is much better suited to hear the case. This dismissal does not prevent a plaintiff from refiling his or her case in the more appropriate forum.

- The issue of Forum Non-Conveniens and Transfer of Venue

There are two types of venue transfer:§ 1404(a) – “For the convenience of parties and witnesses, in the interest of justice, a district court may transfer any civil action to any other district or division where it might have been brought.”

the action was brought in the right place, but there is a more convenient forum to bring it. The substantive law and procedural rules travel with the transfer.

Can only be transferred to a court in which you could bring it on the day the action was filed under either in personam or venue rules.

§ 1406 - The action was brought in the wrong place (no in personam, wrong venue), but there is a right place. None of the law travels.

When a plaintiff brings a change of venue motion the law works the same as if the defendant brought the motion: defendant has to agree

Hill’s Review: Think about public and private factors, jurisdiction, choice of law analysis, forum shopping, rules, private party’s advantages and disadvantages, public party’s advantages and disadvantages.

To get to the fed ct. fed q/diversity = S M Jurisdiction, over the case, applies to fed. courtPJ = J over the parties, mainly ∆ Piper case deals with : SMJ, PJ, VENUEPJ , same rule applies in state or federal court. Exam: go through SMJ, PJ, Venue issues and decide. Think like an attorney, where do you want to go?

Venue is a district by district analysis whereas PJ is state by state Piper: venue and forum non-conveniens (FNC is related to venue)

Negligence or products liability are mostly tort cases, American tort law very lenient hence, popular When it comes to plaintiffs, America: Better products liability, strict liability, law more favorable, punitive damages are betterCalifornia: very liberal tort laws hence makes CA very attractive - US laws more liberal than Scotland Plaintiff’s are forum shopping in a big way.

Federal Question: Federal law applies; Diversity Cases: state law applies

Piper: Why California? No body from United States, decedents including pilot all from Scotland. CA has no real relation apart from the lawyer who is in CA.

∆s have a three prong strategy, P files in state court CA, there is diversity here, but they decline to go to federal court, but ∆s want to go to federal court as once they go to Fed. Ct., they can transfer under 1404 a.

Bates v. C & S - Debtor lived in west.d. of Penn, moves to w.d. of NY C&S debt collection agency, business only in Penn. Fed. Q. Act: Unfair debt collection act Debtor wants to sue under this. This act gives protection if someone harasses you for collection. He sues in NY, C&S raise the venue issue You can’t bring under b1. P. brings it under b2Everything occurred in Penn, letter was sent from Penn to Penn, even when forwarded it wasn't from P. So no substance in NY but court says it was a substantial portion. Amendments to it make the lang. loose. There was no PJ over C&S, there was no aiming at NY, letter going to NY was incidental. It falls under Calder v. Jones.

Piper Aircraft v. Reyno – 1981 (293-303)5 Scottish people died in an airplane crash in Scotland. The plane was American and the pilot flying the plane was Scottish. Lawyer for decedents filed suit against airplane manufacturer in Pennsylvania for wrongful death under negligence and strict liability. D moved for transfer to Scotland based on forum non conveniens.

18

Page 19: Exam Outline Civ Pro

What’s the mechanism of getting into fed. ct. out of state court: they remove (removal) from CA fed court and, then seek transfer to PENN Fed. Court then get it dismissed under forum non conveniens as more easily possible there.

CA fed. court decides that there is jurisdiction over Piper but not Hartzell, they lacked personal juris. over Hartzell, no min. contacts. Piper does business in CA hence there is PJ over Piper. 1404 a permits transfer to PENN, if PENN would have jurisdiction over Hartzell, as they are selling their propellers to Piper in Penn. Within the fed. system transfer is permitted from one state to another, even the case with no jurisdiction as long as the receiving court has jurisdiction.

Doesn't work anymore, but here for our case, Reyno (from CA) was P. (today it will be people from Scotland).Which law would apply? CA - No.

- Penn or Ohio, or Scotland.

Choice of law principles: each state has its own choice of law analysis. Only because you field suit in CA, doesn't mean those laws would apply, other state laws may apply, decided by choice of law analysis.

CA no PJ over Hartzell, only Penn did hence, Penn choice of law principle had to be applied to decide what law would apply According to district court of Penn, for Hartzell - Scottish law as there was where the accident occurred.For Piper, CA had PJ, therefore CA choice of law, they said law will be applied of the place where the article was made, that was Penn But in both these case, CA law won’t apply in either case.

Was Venue proper against Hartzell and Piper? For corporation venue is where there is PJ No PJ over Hartzell, therefore, venue not appropriate for Hartzell but only for Piper as there is personal jurisdiction. Venue is a creature of federal court Piper might be selling directly to CA but not Hartzell, not directly targeting CA, hence no PJ over Piper.

Not enough of Nexus. If accident would have occurred in CA then under Grey, then also there would have been jurisdiction. Hence, this case if different than Grey for Hartzell.

No Nexus to accident here in case of Hartzell.

Forum non conveniens has something to do with venue. Is it used when venue is proper or used when venue is not proper? - It is proper when venue is proper. It is when the court says there is a proper place to do it. Footnotes p.919, factors list: Private

and Public FNC is discretionary.

Essence of these factors? There is this principle that the place where something occurs, the courts have to create an interest in deciding the issue. Here all the decedents were Scottish. Public Factor: which court has the closest ties, greatest interest in being able to adjudicate this. If there are local P.s who have been injured the forum of that place…. the efficiency and rights of the jurisdiction have to be kept in mind.3rd Circuit dismissed because: 1. Scotland was going to be less favorable than U.S. (P.s might not get as much money), but SCOTUS disagreed as even though they may be true, but Scottish law was most favorable. Even if the law is not as generous, there will be some remedy.

P.s have a right to pick their own forum. But the court says, not as much here, why not? P. 923.

Preference is strongest when P.s want to pick their own forum the forum where case is being filed. For ex. if they had been from CA and wanted to pick CA (as it was their own forum) but since they are picking it legally for being more attractive, hence doesn't matter All this is called Forum Shopping.

Foreign P.s look for Nexus in U.S. hence flood of lawsuits here, because of generous laws.

3rd Circuit even change the choice of law analysis to make the case more favorable to make it stay in United States SCOTUS felt that 3rd Circuit was way off.

FNC: discretionary doctrine used by courts to dismiss the case. Used when venue was otherwise technically proper. But in looking at other factors the court says that it really is not the best place for the case to be held.

Decision is made by looking at factors in footnote 6. Also, which city or country has the greatest interest in adjudicating their claims.

Factors: where the evidence is? plane crash in Scotland. Nothing happened in CA, no evidence there.

A. Essence of the Private Factors: where the evidence lie.B. Public Factors : which jurisdiction/sovereign has the greatest interest in adjudicating these claims, in this case Scotland as there is

where everyone is from

What cuts against dismissing? P.s have picked the forum but court says, more credit would have been given had they picked their own forum This was time where FQJ was already growing. Overflowing with litigation. Opening them to foreign litigation was a difficult situation for them.

P.s were forum shopping in a big way.

Why CA? P.s attorney lived and great place for tort law. Principle of P. picking its own forum doesn't get much weight when forum

19

Page 20: Exam Outline Civ Pro

shopping is going on.

20

Page 21: Exam Outline Civ Pro

Exam Review: Identify whether the issue is like hanna or shady grove, is there a federal rule or like byrd, with just a federal policy If policy - 3 parts of balancing test will be waived, don’t worry about fed. rules.If fed. rule on point: is there really a conflict, Scalia says conflict as there is an addition. Ginsberg says no conflict. II step: abridge, enlarge or modifyIs Shady rule: substantive state right?

RDA (Rules Decision Act) -The laws of the several states, except where the Constitution or treaties of the United States or Acts of Congress otherwise require or provide, shall be regarded as rules of decisions in civil actions in the courts of the United States, in cases where they apply.

Erie Doctrine When a federal court is sitting in diversity, state law applies

Diversity (substantive law of a state) Quasi Procedural Procedural (federal courts have their own even when applying state law)- Between substance and process there are “quasi procedural” rules such as SOL and there are therefore issues as to whether federal/state laws should apply to these issues.

Latches – stops a SOL from running – Quasi Procedural

- Fairness issue in forum shopping.

In Swift v. Tyson, contracts past consideration case, is pre-existing debt past consideration? Fed Law – Yes; NY Law – No Story says, common law doctrine applies federal courts need only apply state legislative law -- §34 RDA only applies to these laws- Common law simply evidence of law, not law

Erie case: Common law applies- Here, federal law more lenient towards plaintiff, PA Law – Less LenientCab Case – clear manipulation of rules to take advantage of more favorable federal rule A company reincorporates to create diversity

Erie rules Swift as unconstitutional declares, common law + statutory law must be applied by a federal court in diversity limits vertical forum shopping (state and federal); horizontal shopping, still ok (State A or State B)

- can one get into a state of more favorable laws.the same court sitting in div. should not only use § but state common law as well.

York – Pushes Erie further

Because of “forum shopping”, fed ct. sitting in diversity have to go as far as possible from preventing any differences – ODT If the difference between state and fed. courts can be outcome determinative then state rule should be followed.

- Before Erie, there was a gap, but now we are aligning Fed and State. Not just statute but the common law of state as well. York says, behind Erie, there was a concern about forum shopping. If the difference between state and federal law is outcome

determinative, then State court rule will have to be followed a closely as possible. Erie said we will have to follow state common law

Quasi Proc. Rules - can be outcome det. too.York tried to stick to state laws as closely as possible (Erie on Steroids)

York set up a conflict, fed and state rules may differ.

Swift – StatutesErie – Statutes + Common LawYork – Statutes + Common Law + ODT for quasi procedural

Cohen : Fed. Rule 23 (this rule is how you bring class action). State had a rule, before you can bring class action, Ps had to post a bond. Problem with that is, Fed. Rule 23 did not have that but state rule did, hence, potential conflict, that could be outcome determinative. The state policy requiring bond is in state rule, to prevent strike suits. Based on

Part IIIChoosing the law to be applied “THE ERIE PROBLEM”

Ragan - If we apply the outcome-determinative test to this case, the outcome will definitely be affected if we use federal instead of state rules. Do we follow the state rule in this case? According to York, we must, and that’s what the Supreme Court holds in Ragan.Cohen - The federal rule has no bond. The New Jersey statute says you have to use a bond. Is this outcome-determinative? Yes! Thus, the Supreme Court holds that New Jersey state law trumps FRCP Rule 23.1

21

Page 22: Exam Outline Civ Pro

York, they look at it and decide in Cohen that rule 23 won’t be applied the way not requiring bond, we will follow state procedure and bond will be required. We will have to follow state rule to align state and fed. to prevent forum shopping. Fed. rules can very state to state.

Ragan : statute of limitations stops running under fed. rule 23 when we file the complaint in court. But under state law, statute keeps running until the ∆ is served Court says, following the ODT, here we apply state rule.

Byrd v. Blue Ridge

Issue – guy’s status will determine whether or not he will be immune from liability: If he’s an employee, he’s immune (the employer is liable). If he’s an independent contractor, he’s not. State law says a judge makes this decision. This conflicts with 7 th Amendment, which requires a jury as the finder of fact. "Outcome" is not the only consideration.- did not involve fed. rule of civ. pro, but a conflict, fed. policies that favor letting a jury decide.

- Do we follow the federal or state policy- fed. policy - let jury decide- state policy - let judge decide.

This was the conflict. Fed. Pol. v. State Pol. Was it substantive or procedural? It was more procedural. First case to hold back from ODT of following the state and said there is a conflict btw Fed and State Pol, both are quasi

procedural as to who decides the issue. The litigant’s interest (to avoid ODT) was very important in understanding Byrd. This is a third factor. If there is a diff. bet. fed and state that could be OD, should we go to fed. or state? Byrd gives a balancing test of these three factors: QUASI PROCEDURAL POLICY of letting jury decide. A rule is not involved, it is

an imp. QUASI PROCEDURAL INTEREST. Byrd does not follow the fed. rule of civ. procedure, just a fed. policy v. state policy. Brennan - in fed. ct, fed. policy will be followed. Balancing test 7th Amendment comes into play here. There have to be important state reasons for judge to decide, but under 7th amendment, high value and

importance of jury making these decisions. This case says that (1041, note 2, litigant’s interest…)….two interests on state side, one on fed side.

Hannah v. PlummerP files a lawsuit against deceased D, so he files against D’s estate right before the statute of limitations is up (1 year after D died). P served according to the federal rule (which is codified in the Federal Rules of Civil Procedure), but not the Mass. rule (which is codified in state statute).

The federal rule says you serve someone by giving the notice to the person or someone in their house. The Mass. rule says you file it in the courthouse or you have to personally give it to the executor. P says he followed the Mass. federal rule, and that if he was in state court he would have followed the Mass state court rule. Mass. has an interest in making sure that money in estates is not held up forever. If the executor doesn’t know there is a service out there waiting for him, he cannot distribute the money.

state rule required in hand service. - unlike Byrd, does involve a federal rule.- does appear to be a direct conflict between state and federal rule.

Conclusion: Hannah v. Plummer: follow federal rule, we are in fed. court. Court is backing down on ODT, it says there is a minor difference when it comes to service. There has to be a real significant

difference. Look at the big picture. Court further says that this is not even a ODT type of case, like Byrd or York with a federal rule, but not the same case here.

Rule 4 (d)(1) permits to complaint/summons left with a suitable person, in MA - personal service. The administrator must be personally liable, to protect them MA statute says that you have to serve personally so that he is not blind sided. -

Harlen concurrence. Hanna v. Plummer gives dicta about ODT but decision doesn’t changes. There is this re-shifting of ODT. What happens when we have a fed. rule of civ. pro. in play. What is all that motivated this forum shopping. But for this minor

procedural difference will someone really go to fed. court v. state ct. or vice-versa, this is a minor difference. Does the difference in the rule significant enough (pre-event) to channel into fed. court, “pre-event” issue v. “post-event” issue.

Hannah says that if there is a conflict with federal and state rule, but federal rule is on point then federal rule should be used. Hannah sides with the twin aims of erie: (1) Avoidance of forum shopping, (2) Avoidance of the inequitable administration of the law.

RDA (a section of fed §, when sitting in state, follow the substantive law) follow the substantive law Erie, York, Byrd

REA (§ by which congress gives power to supreme court, the § governing in Hannah) Applies to federal rules of civil procedure

REA: In a court when we have a fed. rule on Pt. we have the REA - where it says follows federal law unless it larges, abridges or modifies a substantive law of state, or does something unconstitutional. REA tracks RDA

- Erie involves Quasi Proc. not governed by the REA. Ginsberg channels Harlen in Shady Grove. Harlen says, this case seems to bend all procedural issues towards fed. rules. Harlen - concerned with protecting sphere of state authority. Scalia is channeling Warren in Shady Grove. - when you are in fed. ct. follow fed. rule. Interpreting Fed. Rule more broadly. going back to Ragan and Cohen

22

Page 23: Exam Outline Civ Pro

Why does state says that SOL keeps running until ∆ is served? If he files the complain he still doesn't know if he is off the hook, it is a way of assuring the ∆. Would justice Warren overrule Ragan? How about Harlen? Does he think that they should follow fed. rule in Ragan? Yes he does. Coehn : in addition to Rule 23, if you have to bring strike suit, you have to post a bond on the state rule. Did Harlen follow fed. rule or would follow state rule? Harlen would have followed the state rule, he argued the case for the ∆. The requirement of a bond is imp. state policy to discourage strike suits. Harlen agrees with Ragan being overruled but not Cohen. Rule in Cohen, substantive state policy, we should be sensitive to that. It’s

not that we disregard federal rule. Ragan and Cohen both tilt is towards the state, not follow the fed. rule, Warren says follow fed. rule, Harlen comes back and splits the

difference. Harlen : states have the authority to pass these “primary” rules. It’s not about procedure but to create a substantive policy that should

be respected.Burlington Northern RR Co. v. Woods

Appealing party in Alabama must pay 10% of the damage if they lose on appeal. The federal rule (38) is that the court can impose up to triple costs at its discretion. Diversity suit is brought in Alabama district court and P wins. D appeals and loses, and P wants his 10%.

Is a federal rule involved here? yes, trying to discourage frivolous appeals. - where the court determines appeal to be bogus, the court can impose additional costs. judge may impose single or double costs, under Fed. Law lot more leeway for the federal judge, he may or may not, single or double. state law : 10% penalty, if judge decides it to me frivolous, the judge must. No discretion. REA applies here as well. More Hannah type case, a rule is on point here. The court follows the federal rule. Whether there was someway of avoiding the conflict? (in Hannah there was a direct conflict.) : The court here says there is a conflict

between fed. and state rule as mentioned above. No way of avoiding it. REA says, the federal rule has to be used unless abridge, enlarge or modify any substantive right….This isn't a substantive right. This is a procedural remedy. It’s a quasi procedural rule to discourage frivolous appeals.

First Q. to ask, is a fed. rule arguably on point? If it’s not what cases do we want to follow?1. It’s not a fed. rule but more quasi procedural : BYRD case should be followed. Balance to three BYRD factors. - the third factor

about litigant’s interest….(stated above).2. If a fed. rule is on point, then we need to ask does it really conflict with the state rule? If yes, go to step 3.

Walker v. Armco, NY has a rule for service of process that says: serve first and then file in court, statute of limitations rule says an action is commenced

when the person is served. Federal court has a rule that says: file first and then serve, statute of limitations says an action is commenced when the papers are filed – Rule 3 of the Federal Rules of Civil Procedure. In diversity case, P files first and the service gets lost behind the radiator and D isn’t served within the statute of limitations.

(it is like Ragan) : Same conflict. State says SOL keeps running until person served, fed. rule, only until complaint filed.Ragan said follow the state rule and so does Walker. Hannah seems to suggest, Ragan bad law but Walker disagrees, says Ragan good law.Why did the court follow the state rule here? Court says no direct conflict between Fed. and State law. Hence, state law is

followed. Walker reaffirms Ragan When isn’t a conflict, follow state rule. If you can't find a way out of the conflict then go to the REA, abridge, enlarge…..line.

23

Page 24: Exam Outline Civ Pro

Shady Grove: 1. Conflict between rule 23 and NY Law which prohibits class actions is suits seeking penalties or statutory minimum

damages. - Good claim made and to be given compensation within 30 days, if not then penalty but NY says not as a class action under

901 (b). $500 penalty amplified to multi-million $ class action is unfair, NY felt. - RULE 23 doesn’t have this. If you meet 4 requirements, you can bring class action. State law doesn’t agree and that is the

conflict. - Scalia: Likes bright line laws. Favors federal law, following Hannah and following Justice warren’s opinion on Hannah.

Says there is a direct conflict. Between New York Law and Fed Law. Rule doesn’t have an additional requirement, we will follow the fed law.

- Ginsberg : No direct conflict. Why? NY law has good substantive reasons like the Cohen case. This is an additional procedural bound up with remedy. No direct conflict. It’s bound up with substantive reasons. State feels unfair to overkill (All State)

- Scalia: Channeling Warren: Not substantive, procedural, no problem under the enabling act. Not an Eerie Problem. - Ginsberg: Channeling Harlen. - For Ginsberg, still an Eerie problem. - Stevens: concurs with majority but takes Halen, Ginsberg approach. - Shady Grove is a lot like Hannah, follow the federal rule and ignore state rule, interpret fed rule very broadly - Burlington: Fed. rule but conflict - Rest all cases (above), state rule.

Mason v. American Fed. Ct. sitting in div. has to apply substantive state law. Mason case: how is that decided? - Legally binding supreme court precedent. If SCt. Precendent not available then lower ct. can be persuasive enough too. - Older rule that changed was never overruled by SCOTUS form 1928. They avoided the issue - This court says, there is a little bit of leeway. - What would the state supreme ct. do today even if there is this precedent going other way. This gives ct. leeway. - Holding: perfectly permissible for fed. ct. to have a leeway, fed. ct. can ignore it if they believe that state sup. Ct. will reach

a different result under some different rule today. - Notes: Fed, Ct. reaching the decision of state law, not binding on state. If fed. ct. makes a decision and circuit ct of appeals

affirms it, then it will be binding on lower cts. Unless the fed. ct. has evidence has state sup. Ct. will do something differently. DICE v. AKRON - The converse erie problem: In erie fed ct sitting in div must apply state subs. law. Here State law applying federal rule. - But where a fed claim can be brought in state ct, state ct has to follow fed law and policy. - Section 1983 fed statute permitting a person to bring a tort like suit when a state officer has violated the constitutional

right, this section permits the victim to sue officer for violation of civil right. If goes into state ct, it will have to fully apply federal law.

- What if there are state defenses that might narrow fed law. Can it be done if congress won’t do it otherwise? In this case, release binding under state law, non-binding under federal law. - Fed law has to be given full effect if congress disagrees with state law, state cannot limit fed law towards state tort defense in this

case. - Who decides? Judge or Jury? Congress says you have to follow the fed law. Congress can require federal procedures to be followed.

APPLICATION TEST 1. Is a federal rule (arguably) on point? a. If no, ---- Byrd If a case with Fed. Procedure, do the Byrd balancing formula: Fed. Int., State Int, Litigant’s interest in avoiding ODT, to avoid

forum shopping. (This third factor weighs in favor of state interest. By following state rule, you are minimizing difference between fed. and state rule.)

- Byrd is a good example of what we do when a conflict btw fed. policy and state policy 2. If yes, does the fed. Rule (FCRP, FRAP) actually conflict with the state rule policy (Ragan, Cohen, Hanna, Plummer,

Burlington, Northern, walker v armco) a. These 6 cases with fed. rule on point. But is there really a conflict btw state and fed. rules, is there a way of avoiding

the conflict? b. In ragan and walker, the court used the state rule. Did they ignore the federal rule? No but narrowed it, found a way of

avoiding the conflict. 3. If there is no conflict, use the state rule (i.e. supplement the fed rule with state rule/policy). 4. If there is a conflict, the federal rule must be applied and the state rule disregarded….unless….. 5. Unless the federal rule “abridges, enlarges or modifies (REA) a substantive rule/policy of the state. Fed rule says judges have discretion, state rule says they do not in Burlington, conflict is unavoidable, we will have to follow fed

rule unless REA makes this allowance for state substantive law. In Burlington it is not a substantive rule/right (10% penalty), it’s procedural. Therefore, no problem with REA definition issue

24